LSAT and Law School Admissions Forum

Get expert LSAT preparation and law school admissions advice from PowerScore Test Preparation.

 Administrator
PowerScore Staff
  • PowerScore Staff
  • Posts: 8917
  • Joined: Feb 02, 2011
|
#58961
Please post your questions below!
 Jbontemps
  • Posts: 3
  • Joined: Nov 04, 2018
|
#60070
Assuming “Most clearly operative” in the question stem means “most apparent” which I took it to mean, how does it follow that what’s most apparent is D? I still don’t quite see that as the best answer choice.

I do admit that that statement would be one that the author might agree with, but MOST apparent or operative...? I chose C, although, albeit “measures should be taken” was a small red flag for me, it overall summarizes the principle there maybe should be state action inhibiting restrictive covenants, which I sensed was the author’s feeling...
 Malila Robinson
PowerScore Staff
  • PowerScore Staff
  • Posts: 296
  • Joined: Feb 01, 2018
|
#60262
Hi Jbontemps,
I would probably rephrase "most clearly operative" into 'most clearly at work' the difference between 'apparent' and 'at work,' to me, is that 'at work' sounds like it is doing something, in this case it is guiding the argument.

The author goes into detail about the issues with the Shelley decision and then states: "Primarily for this reason, neither the (45) Supreme Court nor lower courts later applied Shelley’s approach." This is where answer D comes from.

Answer C has too much strong language in it to match with the tone of the authors argument. We don't know that this is "the most troubling aspect of a practice" and we don't know that any measures that could be taken could 'prevent this practice from continuing in an altered form."
Hope that helps,
-Malila
 IneedLSAThelp
  • Posts: 3
  • Joined: Nov 13, 2018
|
#60402
I got it down to A and D, but chose wrong. Can you please explain why A isn't correct? I interpreted this question as asking: "What principle can be pulled from this passage/What principle is at work in this passage?" This seemed more like one of those principle questions in the arguments sections. Is my understanding of the questions incorrect?
 Malila Robinson
PowerScore Staff
  • PowerScore Staff
  • Posts: 296
  • Joined: Feb 01, 2018
|
#60462
Hi IneedLSAThelp,
Your interpretation of the question seems good (see my explanation above).
Answer A seems to be out of sync with the passage. First, courts failed to use the Shelley decision as precedence, and then the author, who may or may not be a legal scholar, wrote about a confusing aspect of the Shelley decision that could explain other courts' not following the decision. So Answer A seems to be backwards.
Hope that helps!
-Malila
 negins
  • Posts: 3
  • Joined: Nov 15, 2018
|
#60479
I understand why D is correct, but could you please explain why E is wrong? I thought that the author is using the fact that the rationale was problematic to suggest that a new rationale should be provided, which he implies in the last paragraph ("what was troubling was not the covenants' enforcement but their substantive content").
 Malila Robinson
PowerScore Staff
  • PowerScore Staff
  • Posts: 296
  • Joined: Feb 01, 2018
|
#60660
Hi Negins,
Answer E is incorrect because no new evidence is offered to support the original ruling. Instead evidence is offered for why the ruling is confusing. The line you cited may suggest that the ruling would be better with new evidence, but at the moment there isn't any, so the principle is based upon what is there now, not what could be there later.
Hope that helps!
-Malila
 lanereuden
  • Posts: 147
  • Joined: May 30, 2019
|
#65515
Explain why b is wrong please.
 Adam Tyson
PowerScore Staff
  • PowerScore Staff
  • Posts: 5153
  • Joined: Apr 14, 2011
|
#65652
I'm going to put the ball back into your court on this one, lanereuden. Tell us why you think answer B is right. What evidence did you find in the text that supports the idea expressed in answer B? Where do you see that principle operating in the passage? Your answer to every RC question should be based on the text of the passage, and you should always be able to find support for those answers, either directly stated or else strongly implied. It's not what you think might be true or reasonable, but what the passage indicates is true or reasonable.

Get back to us on that, and that will give us a better idea of where you're coming from and how to help you get back on the right course!
 cecilia
  • Posts: 66
  • Joined: Nov 07, 2011
|
#78879
I'll give (B) a try because I also got it wrong. The last half of this answer choice is where it goes off the rails and where I misinterpreted what it actually said.

I thought B was saying that the substantive content should be addressed in a decision (as the passage suggests in L. 11+ 60) But that's not quite what (B) said. Instead of the substantive content being addressed, B says it should be *included* in a statue, which is like, lol, definitely not what the passage was intending.

Get the most out of your LSAT Prep Plus subscription.

Analyze and track your performance with our Testing and Analytics Package.